ارسال ها
40
لایک ها
13
امتیاز
0
پاسخ : بررسی سوالات مرحله دوم المپیاد ریاضی- دوره 32- بهار 1393

اگه شرط m<=n رو بذاریم هم مثال نقض هست براش؟
بله! خودم پیدا کردم مثال نقضشو :|
16 عدد 1000 رو عاد نمیکنه ولی 1000000 رو عاد میکنه! :||
 

سلندر

New Member
ارسال ها
16
لایک ها
2
امتیاز
0
پاسخ : بررسی سوالات مرحله دوم المپیاد ریاضی- دوره 32- بهار 1393

بله! خودم پیدا کردم مثال نقضشو :| 16 عدد 1000 رو عاد نمیکنه ولی 1000000 رو عاد میکنه! :||
راه من: latex.codecogs.com/gif.download?m%3C%20n%5Crightarrow%20n%5E%7Bn%5E%7Bn%7D%7D%3En%5E%7Bn%7D%3Em%5E%7Bn%7D%3Em%5E%7Bm%7D%5Crightarrow%20tanaghoz%5Crightarrow%20m%5Cgeq%20n%20%5Crightarrow%20n%5E%7Bn%7D%3E%20m latex.codecogs.com/gif.download?a_%7Bi%7Dn%5E%7Bn%7D%3Db_%7Bi%7Dm%5Crightarrow%20%5Cfrac%7Bb_%7Bi%7D%7D%7Ba_%7Bi%7D%7D%3D%5Cfrac%7Bn%5E%7Bn%7D%7D%7Bm%7D latex.codecogs.com/gif.download?m%21n%5E%7Bn%7D%5Crightarrow%20vojood%20darad%20%22i%22ke%20%3Aa_%7Bi%7Dn%3Cb_%7Bi%7D%5Crightarrow%20%5Cfrac%7Bb_%7Bi%7D%7D%7Ba_%7Bi%7D%7D%3En%5Crightarrow%20%5Cfrac%7Bn%5E%7Bn%7D%7D%7Bm%7D%3En%5Crightarrow%20m%3Cn%5E%7Bn-1%7D%5Crightarrow%20m%5E%7Bm%7D%3C%28n%5E%7B%5E%7Bn-1%7D%7D%29%5E%7Bn%5E%7Bn-1%7D%7D%3Cn%5E%7Bn%5E%7Bn%7D%7D%5Crightarrow%20tanaghoz وبعد نتیجه می گیریم m توانی صحیح از n است. ببخشید از این مرتب تر نشد.ai و bi به ترتیب توان های pi در n و m اند.
 

mohy1376

Well-Known Member
ارسال ها
418
لایک ها
311
امتیاز
63
پاسخ : بررسی سوالات مرحله دوم المپیاد ریاضی- دوره 32- بهار 1393

من دیگه خسته شدم الآن واقعا کف در جه حدودی است ما بریم سراغ زندگی

راستی چرا mathysc برای من باز نمیشه مال شما هم همینطوره
برای من باز میشه ولی هنوز جوابارو نذاشتن.

همون بهتره که بریم به زندگیمون برسیم.
 

Mehrnaz.T

New Member
ارسال ها
2
لایک ها
1
امتیاز
0
پاسخ : بررسی سوالات مرحله دوم المپیاد ریاضی- دوره 32- بهار 1393

سلام!
میشه بگین سوال 5 قسمت ب رو شما چجوری حل کردین؟!:87:
 
ارسال ها
40
لایک ها
13
امتیاز
0
پاسخ : بررسی سوالات مرحله دوم المپیاد ریاضی- دوره 32- بهار 1393

راه من: latex.codecogs.com/gif.download?m%3C%20n%5Crightarrow%20n%5E%7Bn%5E%7Bn%7D%7D%3En%5E%7Bn%7D%3Em%5E%7Bn%7D%3Em%5E%7Bm%7D%5Crightarrow%20tanaghoz%5Crightarrow%20m%5Cgeq%20n%20%5Crightarrow%20n%5E%7Bn%7D%3E%20m latex.codecogs.com/gif.download?a_%7Bi%7Dn%5E%7Bn%7D%3Db_%7Bi%7Dm%5Crightarrow%20%5Cfrac%7Bb_%7Bi%7D%7D%7Ba_%7Bi%7D%7D%3D%5Cfrac%7Bn%5E%7Bn%7D%7D%7Bm%7D latex.codecogs.com/gif.download?m%21n%5E%7Bn%7D%5Crightarrow%20vojood%20darad%20%22i%22ke%20%3Aa_%7Bi%7Dn%3Cb_%7Bi%7D%5Crightarrow%20%5Cfrac%7Bb_%7Bi%7D%7D%7Ba_%7Bi%7D%7D%3En%5Crightarrow%20%5Cfrac%7Bn%5E%7Bn%7D%7D%7Bm%7D%3En%5Crightarrow%20m%3Cn%5E%7Bn-1%7D%5Crightarrow%20m%5E%7Bm%7D%3C%28n%5E%7B%5E%7Bn-1%7D%7D%29%5E%7Bn%5E%7Bn-1%7D%7D%3Cn%5E%7Bn%5E%7Bn%7D%7D%5Crightarrow%20tanaghoz وبعد نتیجه می گیریم m توانی صحیح از n است. ببخشید از این مرتب تر نشد.ai و bi به ترتیب توان های pi در n و m اند.
ممنون! ولی عکسا نمیاد برای من! آدرسو هم که باز میکنم فقط این میاد: m<n -> n[SUP]n[/SUP]
 

m.qsm

New Member
ارسال ها
6
لایک ها
2
امتیاز
0
پاسخ : بررسی سوالات مرحله دوم المپیاد ریاضی- دوره 32- بهار 1393

اگه شرط m<=n رو بذاریم هم مثال نقض هست براش؟
من اول گفتم n=m=1 درسته بعد برهان خلف زدم که اگه جواب دیگه داشته باشه غلط میشه و ...
اول m , n رو به عوامل اول تجزیه کردم بعد چون عوامل اول m , n مثل همن فقط تواناشون فرق میکنه . با بازی با توان ها و چند تا نامساوی زدن و استفاده از همین نامساوی m<n بدست میاد n^(n-1) <m<n^n و بعد بدست میاد n-1 < 1 که این هم فقط وقتی بدست میاد که n=1 باشه حالا اگه n=1 باشه m=1 میشه که ما از اول فرض کردیم که اینا نباشن پس به تناقض رسیدیم
 

asmath

New Member
ارسال ها
8
لایک ها
0
امتیاز
0
پاسخ : بررسی سوالات مرحله دوم المپیاد ریاضی- دوره 32- بهار 1393

اولا که من خودم اثبات کردم حداکثرش بعد از وقت هایی که چندتا عدد یکسان ۱ حذف می شن رخ نمیده دوما سوال گفته ثابت کنید از یه زمانی به بعد حداکثر داره...

---- دو نوشته به هم متصل شده است ----
خوب دوست عزیز منهم که گفتم که یه جایی به چند تا عدد متوالی می رسیم خوب از همون جا حد اکثر داره دیگه مشکلش چیه


ب
 

ash1374

New Member
ارسال ها
253
لایک ها
422
امتیاز
0
پاسخ : بررسی سوالات مرحله دوم المپیاد ریاضی- دوره 32- بهار 1393

روز دوم بر خلاف روز اول امتحان نسبتا سختی بود. و اینکه هندسه نداشت!!! امتحان از نظر حواشی شبیه سال 88 بود:4:. سوال شش سوال زیبایی بود. از سوال 5 اصلا خوشم نیومد. نتونستم بیشتر از ده دقیقه روش فکر کنم. سوال چهار هم نسبت به چهار بودنش واقعا سخت بود.

سوال 4) به استقرا روی k ثابت کنید اگر k بین 0 تا n-1 باشه عدد طبیعی a وجود داره که :
( پایه ی استقرا همون فرض مسئله میشه)
بعد قرار بدین k=n-1 و حکم رو نتیجه بگیرید.
سوال 6) فرض کنید
بزرگ ترین عدد مثلثی کمتر از n باشه. به استقرا روی k ثابت کنید اگر k عددی کمتر مساوی n باشه بعد از
مرحله به این اعداد میرسیم :


ضمن احترام به بچه های تهرانی امیدوارم تعداد خوبی شهرستانی قبول بشن، حداقل بیش از سه نفر که پارسال قبول شدن(هر چند با این امتحان بعیده!)

موفق باشید
 

asmath

New Member
ارسال ها
8
لایک ها
0
امتیاز
0
پاسخ : بررسی سوالات مرحله دوم المپیاد ریاضی- دوره 32- بهار 1393

خواهشا یکی جواب منو بده اگر به آخر سوال نرسیم و نتیجه گیری های آخرش رو ننویسیم امکان داره نمره ای بگیریم؟اگه داره چه قدر احتمالا نمره می گیریم؟با تشکر
 

AMIR0123

New Member
ارسال ها
12
لایک ها
6
امتیاز
0
پاسخ : بررسی سوالات مرحله دوم المپیاد ریاضی- دوره 32- بهار 1393

حالا این امکان وجود داره که حذف کنن ؟ تا قبل از مرحله دو هی فرانسوی ها و آلمانی ها رو مسخره می کردم که میرن سوالای کشورهای دیگه رو بر میدارن اما انگار این مشکل تو کشور خودمون هم هست.
خدا کنه که حذفش کنن چون واقعا ظلم بزرگی هست به افرادی که تا حالا این سوال رو ندیدن .
البته که میشه...نمونش سوال 3 سال 1383 که مثال نقض داشت و باشگاه حذفش کرد و نمرش بین سایر سوال ها تقسیم شد...

---- دو نوشته به هم متصل شده است ----

منظورتون اینه که این سوال هم مث سوال 3 قبلا جایی مطرح شده بوده؟؟ :43:
دقیقا چندتایی از بچه های کامپیوتر به ما گفتن که مشابه این سوال بهشون تو کلاسای المپیاد گفته شده...

---- دو نوشته به هم متصل شده است ----

دوستان یه سوال
من تو سوال اول جوابم اینه:
بدترین حالت اینه که در یک سبد کالا تمامی برنج ها و تخم مرغ ها باشه و بقیه خالی باشن.پس ما 1 سبد کالا پر و 99 سبد کالا خالی داریم.
در هر مرحله ما یه سبد خالی و اون سبد پر رو انتخاب میکنیم و سبد کالا خالی رو به اندازه لازم پر میکنیم.
پس ما طی 99 حالت تمام سبد کالا ها رو درست کردیم.
--------
نمره میگیرم؟؟؟؟
عموما تو مسایل اثباتی نباید از واژه های بدیهی و بدترین یا بهترین حالت استفاده بشه...مگه این که اثبات کنی...
 
ارسال ها
16
لایک ها
20
امتیاز
0
پاسخ : بررسی سوالات مرحله دوم المپیاد ریاضی- دوره 32- بهار 1393

آقا من یه چی بگم یه نمه شاد شید بعد مرحله 2 !!!!!!!!!!!!!!!! من روز دوم 2/75 حل کردم یا حتی 3 تا ولی روز اول کامل هیچی حل نکردم و دوره بره 10 نمره می گیرم و تو کل آزمون نمرم حدودا" 28 - 29 میشم در ضمن دوم هستم نظرتون راجبه من چیه ؟؟؟؟؟؟؟؟؟؟؟؟؟؟؟؟؟؟؟؟؟ قبول میشم ؟؟؟؟؟؟؟؟؟؟؟؟؟؟
 
آخرین ویرایش توسط مدیر

AMIR0123

New Member
ارسال ها
12
لایک ها
6
امتیاز
0
پاسخ : بررسی سوالات مرحله دوم المپیاد ریاضی- دوره 32- بهار 1393

من با استفاده از اعداد مثلثی رفتم ولی جواب با اونی که سوال گفته بود ثابت کنید فرق داشت. من در آوردم


---- دو نوشته به هم متصل شده است ----

من با استفاده از اعداد مثلثی رفتم ولی جواب با اونی که سوال گفته بود ثابت کنید فرق داشت. من در آوردم
من هم همینو بدست آوردم باید به علاوه 1 کنی بعد ثابت کنی از رادیکال 2n به علاوه 1 بیشتر مساویه...

---- دو نوشته به هم متصل شده است ----

در حق شهرستانی ها واقعا ظلم میشه با اون همه کلاس که تهرانی ها دارن باید هم قبول شن!!
بله و این مشکل باید از ریشه حلشه...باید به همه امکانات یکسان بدن...خیلی تو همین شهر ما هستن که میخوان برای المپیاد خرج کنن اما تنها چیزی که دور و برشونه کتابه و خودشون

---- دو نوشته به هم متصل شده است ----

میدونستم! ولی پرسیدم، بلکه یکی بهم امید کاذب بده!!
---
راستی سوال 4 به جواب رسیدم ولی هیچ قضیه ای رو اثبات نکردم!! نمره خیلی کم میکنن؟!
کلا همه چیز باید اثبات بشه...حتی برای ثابت نشدن این که n^2+1 مربع کامل نیست (n طبیعی) از یک نفر نمره کم کردن
 

Mostafa_

New Member
ارسال ها
527
لایک ها
445
امتیاز
0
پاسخ : بررسی سوالات مرحله دوم المپیاد ریاضی- دوره 32- بهار 1393

من روز اولو خوب دادم ... امروزو خيلي بعد خراب كردم .... سوال يك اين خلاصه راه حلمه:

اول گفتم كه اگه m از n كمتر باشه جواب نداره ... اگه m و n برابر باشه جواب m=n=1 هست ... اگر m بزرگتر از n به توان n باشه باز هم جواب نداره حالا حالتي رو بررسي كردم كه m بين n و n به توان n باشه ..... تو اين حالت تجزيه m و n به اعداد اولو نوشتم بعد گفتم عوامل m و n بايد برابر باشه .... بعد اثبات كردم تمام توانايي عوامل اول m از n بيشتره .... تا همينجا ادامش بايد مي نوشتم پس m عاد مي كنه n رو در نتيجه بايد كوچكتر مساوي n باشه ولي اولش فرض كرديم از n بايد بزرگتر باشه ژس تو اين حالت جوابي نداره ...... چند نمره شو مي گيرم؟

سوال 5 من حواسم نبود راه حل + مثالمو براي اعداد حقيقي نوشتم نه اعداد حقيقي مثبت .... ايده اصليشم اين بود اگه بخواد بيشتر از 4 تا باشه و توانا باشه بايد S نامتناهي باشه كه خلافه فرضه ...... اين رو كلا هيچ نمره اي نمي گيرم؟؟؟؟؟؟؟؟؟؟؟؟؟؟؟
 

AMIR0123

New Member
ارسال ها
12
لایک ها
6
امتیاز
0
پاسخ : بررسی سوالات مرحله دوم المپیاد ریاضی- دوره 32- بهار 1393

دوستان
من تو سوال 4 فقط تو پاسخنامه این رو نوشتم.
n=m=1
هیچی دیگه ننوشتم.
چند نمره میگیرم؟
0
البته اگه مصحح خیلی مهربون باشه 1 ولی بازم فکر نکنم...
 

mahdi math

New Member
ارسال ها
152
لایک ها
61
امتیاز
0
پاسخ : بررسی سوالات مرحله دوم المپیاد ریاضی- دوره 32- بهار 1393

دوستان من سوال 4 رو اینجوری نوشتم که عدد اولی که عضو mوn هست به همین ترتیب تا با راه حل جبری به تناقض در تساوی رسیدم درست میتونه باشه در ضمن در مورد سوال 1 گفتم اگه دو سبد پیدا بشه که تعداد تخم مرغ>60 , برنج>20 آنگااه جواب 99 ولی من نتونستم اثبات کنم اما شبیه راه حل اصلی رفتم قبوله؟ چند میدن ؟:205:

---- دو نوشته به هم متصل شده است ----

من که دیگه دارم نا امید میشم...کلا سه تا حل کرده بودم ه یکیشم می لنگه برم خودکشی کنم خلاص شم :( ﷼*مرگ بر المپیاد*﷼
آخ گفتی منم دقیقا مثل تو !!!!!!
 

AhmadrezaBM

New Member
ارسال ها
213
لایک ها
40
امتیاز
0
پاسخ : بررسی سوالات مرحله دوم المپیاد ریاضی- دوره 32- بهار 1393

من هم همینو بدست آوردم باید به علاوه 1 کنی بعد ثابت کنی از رادیکال 2n به علاوه 1 بیشتر مساویه...
آخه چرا اینجوریه؟ بیان تو سوال همون عدد واقعی شو بگن دیگه. مگه نه اینه که جواب درستش همونیه که ما اول در آوردیم. حالا به ما نمره کاملو میدن؟؟؟ اگه ندن نامردیه...
 

aras2213

New Member
ارسال ها
216
لایک ها
228
امتیاز
0
پاسخ : بررسی سوالات مرحله دوم المپیاد ریاضی- دوره 32- بهار 1393

آخه چرا اینجوریه؟ بیان تو سوال همون عدد واقعی شو بگن دیگه. مگه نه اینه که جواب درستش همونیه که ما اول در آوردیم. حالا به ما نمره کاملو میدن؟؟؟ اگه ندن نامردیه...
ببخشید من راه حل شما و اون دوست عزیز رو نفهمیدم اگر ثابت کنیم اون عدده که شما بدست اوردید از اون چیزی که مساله می خواد بیشتره حل میشه؟ چون که سوال گفته حداکثر ...اشتباه می کنم آیا؟ میشه یکم بیشتر توضیح بدید؟
 

AMIR0123

New Member
ارسال ها
12
لایک ها
6
امتیاز
0
پاسخ : بررسی سوالات مرحله دوم المپیاد ریاضی- دوره 32- بهار 1393

آخه چرا اینجوریه؟ بیان تو سوال همون عدد واقعی شو بگن دیگه. مگه نه اینه که جواب درستش همونیه که ما اول در آوردیم. حالا به ما نمره کاملو میدن؟؟؟ اگه ندن نامردیه...
چون ایده اصلی حل سوال رو مطرح کردی 5 تا 7 نمره میگیری...
 

sigma

New Member
ارسال ها
21
لایک ها
5
امتیاز
0
پاسخ : بررسی سوالات مرحله دوم المپیاد ریاضی- دوره 32- بهار 1393

سلام!
میشه بگین سوال 5 قسمت ب رو شما چجوری حل کردین؟!:87:
من برای مثال حالت کلی یک مجموعه توانا رو بدست اوردم بعدش اثبات کردم که اگر عضوی اضافه یا - میشه یا ۰ یا تو خودشه یا اینکه دیگه توانا نمی مونه.حالت کلی هم n,1/n ,1 و رادیکال n به فورجه n بود!

---- دو نوشته به هم متصل شده است ----

سلام!
میشه بگین سوال 5 قسمت ب رو شما چجوری حل کردین؟!:87:
من برای مثال حالت کلی یک مجموعه توانا رو بدست اوردم بعدش اثبات کردم که اگر عضوی اضافه یا - میشه یا ۰ یا تو خودشه یا اینکه دیگه توانا نمی مونه.حالت کلی هم n,1/n ,1 و رادیکال n به فورجه n بود!
 

Mostafa_

New Member
ارسال ها
527
لایک ها
445
امتیاز
0
پاسخ : بررسی سوالات مرحله دوم المپیاد ریاضی- دوره 32- بهار 1393

من روز اولو خوب دادم ... امروزو خيلي بعد خراب كردم .... سوال يك اين خلاصه راه حلمه:

اول گفتم كه اگه m از n كمتر باشه جواب نداره ... اگه m و n برابر باشه جواب m=n=1 هست ... اگر m بزرگتر از n به توان n باشه باز هم جواب نداره حالا حالتي رو بررسي كردم كه m بين n و n به توان n باشه ..... تو اين حالت تجزيه m و n به اعداد اولو نوشتم بعد گفتم عوامل m و n بايد برابر باشه .... بعد اثبات كردم تمام توانايي عوامل اول m از n بيشتره .... تا همينجا ادامش بايد مي نوشتم پس m عاد مي كنه n رو در نتيجه بايد كوچكتر مساوي n باشه ولي اولش فرض كرديم از n بايد بزرگتر باشه ژس تو اين حالت جوابي نداره ...... چند نمره شو مي گيرم؟

سوال 5 من حواسم نبود راه حل + مثالمو براي اعداد حقيقي نوشتم نه اعداد حقيقي مثبت .... ايده اصليشم اين بود اگه بخواد بيشتر از 4 تا باشه و توانا باشه بايد s نامتناهي باشه كه خلافه فرضه ...... اين رو كلا هيچ نمره اي نمي گيرم؟؟؟؟؟؟؟؟؟؟؟؟؟؟؟
l


از دوستانی که واردن یکی جواب منو بده لطفا
 
بالا